Mark has already run 33.4 miles this week. He can run 2.5 miles in one hour. Construct an equation that represents the total number of miles run based on how many hours he runs for. What is the total number of miles he ran if he runs for 17 hours?

Answers

Answer 1

Answer:

y = 2.5x + 33.4, y = 75.9 miles

Step-by-step explanation:

Equation is y = 2.5x + 33.4

17 = 2.5 x + 33.4

y = 2.5 (17) + 33.4

y = 42.5 + 33.4

y = 75.9 miles


Related Questions

Here’s another one for you people

Answers

Brianna’s Roofing Service can build 50 homes and still have 200 boxes of shingles left.

“ Use what you know about function notation to complete the statements “ PLS HELPP

Answers

Step-by-step explanation:

I don't know the options to pick from.

the 2 main possibilities coming to my mind are

y = f(x)

or

(x, f(x)) or (x, y)

there is no specific function coming to mind. the functional values bounce around above and below the x-axis (positive and negative y-values) with not detectable pattern related to the x values.

the other 2 questions can be found directly in the table :

f(3) means f(x) with x=3, and that is -2

and f(x) = -5 is true, when x = 4

Hugo is mixing blue paint with red paint to create purple paint.
The ratio of blue to red is 3 to 2.

How many pints of blue paint will he use to create 40 pints of purple paint?

Answers

Answer:

24 pints of blue paint. I don't know of this is a solid way to get the answer, but I added 3+2=5. then 40/5=8. Then multiplied 8*3=24 and 2*8=16

Step-by-step explanation:

Hassan surveyed 120 of the students in his school about their favorite color. 114 students said their favorite color was purple. What percentage of the surveyed students said their favorite color was purple?

Answers

Answer:

95%

i think it should be that

5% because you take the 6 that was the difference and then you divide it by the whole and then you multiply the decimal by 100 to get 5%

Lin has a pet German Shepherd that weighs 38 when measured in one unit and 84 when measured in a different unit. which measurement is in pounds, and which is in kilograms? ​

Answers

Answer:

38= kilo 84= pounds

Step-by-step explanation:

What can you multiply by to find:

50% of a number?
10% of a number?
75% of a number?

Answers

Answer:

number · 0.50 = 50% of a number

number · 0.10 = 10% of a number

number · 0.75 = 75% of a number

Step-by-step explanation:

50% = 0.50

10% = 0.10

75% = 0.75

Q.

10² + 20² - 5! = ?

note : ! it's a factorial sign

Goodluck​

Answers

[tex] = 10 {}^{2} + 20 {}^{2} - 5! \\ = 100 + 400 - \\ (5 \times 4 \times 3 \times 2 \times 1) \\ = 100 + 400 - 120 \\ = 500 - 120 \\ = 380[/tex]

Question:-

10² + 20² - 5! = ?

Answer:-

380

Explanation:-

=> 10² + 20² - 5!

=> 100 + 400 - (5 × 4 × 3 × 2 × 1)

=> 100 + 400 - 120

=> 500 - 120

=> 380

A pump fills a pool at a constant rate. At the end of 4 minutes it has filled 36 gallons
of water. Which table represents the relationship between the number of minutes and
the number of gallons of water in the pool?
А
B
Time
Water
(minutes) (gallons)
Time Water
(minutes) (gallons)
4.
1
1
4
8
2
2
8
12
3
3
12
с
D
Time
Water
(minutes) (gallons)
Time Water
(minutes) (gallons)
1
9
9
1
2
18
18
2
3
27
27
3

Answers

Answer:

C

Step-by-step explanation:

36 gallons ÷ 4 minutes = 9 per minute

Colton and Gage were building a castle together. Colton was 5 times as fast at building the castle than Gage. If it takes 120 blocks to build the castle, how many blocks did Colton build on the castle?
A. 144 blocks
B. 100 blocks
C. 24 blocks
D. 20 blocks

Answers

Answer:

100 blocks.

Step-by-step explanation:

Let x represent number of blocks built by Gage.

We have been given that Colton was 5 times as fast at building the castle than Gage, so number of blocks built by Colton would be .

We are also told that it takes 120 blocks to build the castle. Since Colton and Gage were building a castle together, so number of blocks built by both will be equal to 120.

Hope it helped!

help was i correct please help for brainlist!!!!!!!!!!!!!!

Answers

You're correct! we can see the population is not going down by half each time, so that eliminated b and c. Since we start with 5000, this makes c the right answer.

Hope this helps :)

Answer: c

Step-by-step explanation:

Is 1:4 equivalent to 12:3

Answers

Answer: no

Step-by-step explanation:

first, we have to simplify when finding if they are equivalent or not.

1:4 = 0.25

12:3 = 4:1

Answer:

Yes (if your doing fractions)

Step-by-step explanation:

If you didn’t know how to simplify you could see the greatest possible multiple that 3 and 12 share. So 3 is the greatest possible multiple because 3x4=12. So you do the denominator and numerator to divide 3 and eventually you will get 1/4. (Sorry if I’m not accurate)

Can someone please help me with this question

Answers

Step-by-step explanation:

please mark me as brainlest

If anyone reading this is in K12, what state are you in?
I do FL

Answers

Answer:

I am I the state of California

20 % of 2 is equal to
A. 20
B. 4
C. 0.4
D. 0.04

If Log 4 (x) = 12, then log 2 (x / 4) is equal to
A. 11
B. 48
C. -12
D. 22

Answers

Answer:

the answer to the first question is c. the second one is

2

\frac{x}{2}

2x​

x/2

Step-by-step explanation:

Answer:

20% of 2 is equal to 0.4

log 2 (x / 4) is equal to 22.

3,415.72 - 1,838.28 = 1577.44
4. Janet earns $6.25 an hour for guarding paintings. If she works 24.3 hours a week for a period of six
weeks, how much does she earn from working six weeks?

Answers

$911.25.

6.25x24.3= 151.875
151.875x6=911.25

Please, Please help ASAP

Answers

Answer:

5 is the answer

please mark as brainlist

thank you

Find a direct relationship
between y and x.
x= e2t and y= et
y = [?

Answers

Answer:

y=√x

Step-by-step explanation: here are the steps

Wort out: 8 divided by 2/3

Answers

Answer:12

Step-by-step explanation:

You have $10 in your bank account. It doubles in the first month and every month thereafter. How much money will you have after 5 months?

Answers

after 5 months you will have $160

so,
10
10×2 =20
20×2=40
40×2 =80
80×2=160
hence proved that after 5 months you will have $160

How long is a bus journey that starts at 8.55 a.m. and ends at 9.17 a.m.?

Please can you put the answer in minutes?

Answers

Answer:

5+17=22 minute

!!!NEED ANSWERS ASAP!!!! simplify 15 ÷ 3 +2 x 10

Answers

Answer:

70

Step-by-step explanation:

because 15 divided by 3 is 5+2=7 and 7 x 10=70

Simplify the question

Answers

I will explain the steps later:)

A group of 120 seventh- and eighth-grade students were asked whether they participate in sports or in nonâ€"sports-related activities. The two-way table shows their answers. Participation in Sports or Non-Sports A 4-column table with 3 rows. The first column has no label with entries seventh grade, eighth grade, total. The second column is labeled sports with entries 39, 29, 68. The third column is labeled non-sports with entries 24, 28, 52. The fourth column is labeled total with entries 63, 57, 120. Which statement is correct? Twenty-nine seventh-grade students chose a sports activity. Twenty-eight eighth-grade students chose a sports activity. Twenty-four seventh-grade students chose a non-sports activity. Thirty-nine eighth-grade students chose a non-sports activity.

Answers

The true statement about the group of 120 seventh-and eighth-grade students is (c) twenty-four seventh-grade students chose a non-sports activity

The entries are given as:

                           Sports       Non Sports          Total

Seventh Grade:     39              24                    63

Eighth Grade:        29              28                    57

Total                       68              52                   120

From the above entries, we have the following highlights:

39 seventh-grade students chose a sports activity. 29 eighth-grade students chose a sports activity. 24 seventh-grade students chose a non-sports activity28 eighth-grade students chose a non-sports activity.

From the above highlights, we can conclude that the following statement from the options is true

(c) twenty-four seventh-grade students chose a non-sports activity

Read more about data representation at:

https://brainly.com/question/1592223

Answer:

c)

Step-by-step explanation:

Write an equation in point-slope form of the line
that passes through the points (5. – 1) and
(-6,-4).

Answers

Answer:

y+1 = [tex]\frac{3}{11}[/tex](x-5)

Step-by-step explanation:

First, find the slope.

-4--1= -3

-6-5= -11

3/11 is the slope

Plug in the slope and one the the points.

y+1 = [tex]\frac{3}{11}[/tex](x-5)

Follow me on  i n s t a g r a m, send a message, and I'll follow you back : ifaulkner24

If a coordinate point is at (4, 2) and is translated down 3 units and right 1 unit, what is its new coordinate point? *

Answers

Answer:

1,3

Step-by-step explanation:

If a coordinate point is at (4, 2) and is translated down 3 units and right 1 unit, what is its new coordinate point?

4-3=1

2+1=3

(1,3)

Answer: (-1,5)

Step-by-step explanation:

Please help meeeeeeee! Gratefully appreciate it, happy holidays!

Answers

Answer:

x = 50

Step-by-step explanation:

180-(50+80)=x

180-130=50

heda ate 1/12 box of ceral now the box is 3/4 full

Answers

The answer would be 5/4

A donut shop made 50 donuts. They put frosting on 33 of the donuts. What is the ratio of the number of donuts with frosting to the total number of donuts?

Answers

Answer:

33:50

Step-by-step explanation:

if there are 50 donuts that gives you 1 part of your answer. Then you have 33 donuts frosted there the other part. Glad I can help

Christian's mom made some cookies. Christian ate 3\9 of the cookies and gabriel ate 1\3 of them. What fraction of the cookies were left?

Answers

Answer:

1/3 of the cookies remain

Step-by-step explanation:

Christian ate 3/9 of the cookies, which can be simplified into 1/3

Gabriel ate the other 1/3

Therefore, there are only 1/3 of the cookies remaining

please help.
with the steps

Answers

Answer:

  -4/243

Step-by-step explanation:

Your calculator can do this.

You know that 3³ = 27 and (-3)³ = -27. You also know that 3² = 9, and that anything to the zero power is 1. With these facts, you can simplify the expression to ...

  [tex]\dfrac{\sqrt[3]{-27}-27^0}{3^3\times3^2}=\dfrac{-3-1}{27\times9}=\boxed{-\dfrac{4}{243}}[/tex]

Other Questions
What type of reasoning is being used?Elphaba was getting very angry and frustrated because none of her good deeds were beingrecognized as good. Causal reasoningInductive reasoningDeductive reasoningAnalogical reasoning Six movie tickets cost $96 what is the cost per ticket ChenAn architect designs two similar triangular patos. The first patio has angle measures of (x-30), (y+15), and 60 The second patio has angle measures of X -10 40 and 80 Find the equation of the line passing through the points (-1,7) and(2,-8). Write the equation in slope-intercept form.A) y = -5x + 2B) y = -5x + 7C) y = 5x + 12 D) y = 5x 18Please help All of the following were colonized by France EXCEPTa) Louisianab) Algeriac) Vietnamd) Cubae) Quebec please help! ASAP I need help fast!! What is the value of x and y? What is a consumer product? if 18 plums weigh 54 ounces then 27 plums weigh x ounces I am DESPERATE!!! PLEAAASE!! I WILL DIEEEEtoy train blocks made of 6 inches long or 7 inches long can be hooked together to make longer trains. which of the following train-lengths cannot be by hooking together either 6 inch tran cars, 7 inch cars, or a combination of both: 29 inches, 30 inches, 31 inches, 32 inches, 33 inches Differences between socialism and capitalism Which chemical contains the greatest store of free energy in one molecule? A. ATP B. glucose C starch D. chlorophyll What happens to the temperature of an object when the particles are moving faster. I cannot comment pls help me When magma cools _____, the crystals it forms are small. When magma cools _____, the crystals it forms are large. quickly; slowlyslowly; quicklyslowly; very slowlyrapidly; very quickly Why did the principles of beneficence and autonomy sometimes conflict with each other May I please receive help? The value of 63 is between what two numbers?VO 62 and 64O 31 and 32O 7 and 80 3 and 4 mai has 36$ to spend on movie tickets each movie ticket cost 4.50$ how many ticket can she buy multiplication problem Determine which postulate or theorem can be used to prove that ASEA = A PEN E O A. ASA OB. SSS O C. AAS O D. SAS